Lesson: Weaken the Argument Questions

Comment on Weaken the Argument Questions

these courses are amazing--are there any PDF handouts available by chance?
gmat-admin's picture

Thanks for the kind words. I have several downloadable flashcards. They (and other links) can be found here: https://www.gmatprepnow.com/content/free-content

Hi Brent,

Could you please update the link for the flashcards.

Thank you,
Ahmed
gmat-admin's picture

Done! Thanks for the heads up!

I cant thank you enough for these videos. Your explanations are quite good but, i couldn't answer any question correctly that were given in the Related Content. More over i am taking close to 6 minutes trying to answer questions with more words/text. Do you have any advice for students as me, please.
gmat-admin's picture

The strategy for solving this question type is very straightforward. In fact, you'll find that most GMAT resources recommend similar strategies for solving them.

You might want to revisit the lesson on Dissecting Arguments (https://www.gmatprepnow.com/module/gmat-critical-reasoning/video/1133), and just spend some time analyzing CR passages to identify premises and conclusions. Other than that, just keep working at these questions.

Thanks for your advice, I will keep practicing.

Brent, could you help me on this below question? It is a little bit confuse for me, and in the GMAT Club Forum there are people who choiced C, D and E. I choiced the letter E.

In the United States, of the people who moved from one state to another when they retired, the percentage who retired to Florida has decreased by three percentage points over the past ten years. Since many local businesses in Florida cater to retirees, these declines are likely to have a noticeably negative economic effect on these businesses and therefore on the economy of Florida.

Which of the following, if true, most seriously weakens the argument given?

(A) People who moved from one state to another when they retired moved a greater distance, on average, last year than such people did ten years ago.

(B) People were more likely to retire to North Carolina from another state last year than people were ten years ago.

(C) The number of people who moved from one state to another when they retired has increased significantly over the past ten years.

(D) The number of people who left Florida when they retired to live in another state was greater last year than it was ten years ago.

(E) Florida attracts more people who move from one state to another when they retire than does any other state.

Thanks!
gmat-admin's picture

Here's my response to the question: https://gmatclub.com/forum/in-the-united-states-of-the-people-who-moved-...

Cheers,
Brent

Brent,

I have another doubt. The below question I am confused between the answer A and the answer C. In the GMAT Forum some people choiced A, while others choiced C. Could you help me please?

Business are suffering because of a lack of money available for development loans. To help businesses, the government plans to modify the income-tax structure in order to induce individual taxpayers to put a larger portion of their incomes into retirement savings accounts, because as more money is deposited in such accounts, more money becomes available to borrowers.

Which of the following, if true, raises the most serious doubt regarding the effectiveness of the government's plan to increase the amount of money available for development loans for businesses?

(A) When levels of personal retirement savings increase, consumer borrowing always increase correspondingly

(B) The increased tax revenue the government would receive as a result of business expansino would not offset the loss in revenue from personal income taxes during the first year of the plan.

(C) Even with tax incentives, some people will choose not to increase their levels of retirement savings.

(D) Bankers generally will not continue to lend money to businesses whose prospective earnings are insufficient to meet their loan repayment schedules.

(E) The modified tax structure would give all taxpayers, regardless of their incomes, the same tax savings for a given increase in their retirement savings.

Thanks!
gmat-admin's picture

Brent, why this below question is not the answer B? Could you explain me please?

Guidebook writer: I have visited hotels throughout the country and have noticed that in those built before 1930 the quality of the original carpentry work is generally superior to that in hotels built afterward. Clearly carpenters working on hotels before 1930 typically worked with more skill, care, and effort than carpenters who have worked on hotels built subsequently.

Which of the following, if true, most seriously weakens the guidebook writer’s argument?

(A) The quality of original carpentry in hotels is generally far superior to the quality of original carpentry in other structures, such as houses and stores.
(B) Hotels built since 1930 can generally accommodate more guests than those built before 1930.
(C) The materials available to carpenters working before 1930 were not significantly different in quality from the materials available to carpenters working after 1930.
(D) The better the quality of original carpentry in a building, the less likely that building is to fall into disuse and be demolished.
(E) The average length of apprenticeship for carpenters has declined significantly since 1930.

Thanks

Hi Brent,

I was confused between D and E, and chose D. Can you explain me why D is wrong.

https://gmatclub.com/forum/the-introduction-of-new-drugs-into-the-market-is-frequently-220355.html
gmat-admin's picture

Question link: https://gmatclub.com/forum/the-introduction-of-new-drugs-into-the-market...

PREMISE: shortage of human subjects prevents new drugs on market

PREMISE: lives/health of FUTURE generations depend on treatments that are currently experimental

CONCLUSION: physicians morally wrong when, in absence of existing treatment, fail to encourage patients to volunteer for trials.

Answer choice D says "An experimental drug cannot legally be made available to patients unless those patients are subjects in clinical trials of the drug"

How does this weaken the conclusion that physicians are morally wrong when the don't encourage patients to volunteer for trials?

If anything it strengthens the argument, because it suggests that doctors SHOULD encourage their patients to participate in the trials to get the drugs.

Conversely, answer choice E says "Physicians have an overriding moral and legal duty to care for the health and safety of their current patients."

That is, doctors are NOT obliged to ensure the health of FUTURE patients (future patients who will benefit from the tested drugs). Instead, the doctors are obliged to ensure the health of their PRESENT patients, and if there are potential dangers involved in these clinical trials, then the doctor should NOT encourage his/her patient(s) to participate.

Hi Brent-

Please help with below question, I selected ans. choice B.

Also please confirm is it weaken or strengthen the argument question?
https://gmatclub.com/forum/editorial-the-roof-of-northtown-council-s-equipment-storage-48863.html
gmat-admin's picture

Question link: https://gmatclub.com/forum/editorial-the-roof-of-northtown-council-s-equ...

This is actually a Strengthen the Argument question. I placed it in the wrong place. I've now moved it to correct location.

Incidentally, here's my solution: https://gmatclub.com/forum/editorial-the-roof-of-northtown-council-s-equ...

ASIDE: If this were actually a Weaken the Argument question (since I did have it in a long list of Weaken the Argument questions), then B would have been the best answer (as you have chosen).

Hi Brent- is it sufficient if we only focus on Reinforcement activities given at the end of each CR lesson and Practice Questions from Various Official Guides given at the end of SC module?
gmat-admin's picture

Yes, that should be a sufficient amount of practice.

Hello Brent

Could you help provide the answer to the question below:

In the past, most children who went sledding in the winter snow in Verland used wooden sleds with runners and steering bars. Ten years ago, smooth plastic sleds became popular; they go faster than wooden sleds but are harder to steer and slow. The concern that plastic sleds are more dangerous is clearly borne out by the fact that the number of children injured while sledding was much higher last winter than it was 10 years ago.

Which of the following, if true in Verland, most seriously undermines the force of the evidence cited?

(A) A few children still use traditional wooden sleds.
(B) Very few children wear any kind of protective gear, such as helmets, while sledding.
(C) Plastic sleds can be used in a much wider variety of snow conditions than wooden sleds can.
(D) Most sledding injuries occur when a sled collides with a tree, a rock, or another sled.
(E) Because the traditional wooden sleds can carry more than one rider, an accident involving a wooden sled can result in several children being injured.

Hi Brent, can you help deconstruct the argument below?

Members of the Amazonian Akabe people commonly take an early-morning drink of a tea made from the leaves of a forest plant. Although they greatly enjoy this drink, at dawn they drink it only in small amounts. Anthropologists hypothesize that since this tea is extraordinarily high in caffeine, the explanation for the Akabe’s not drinking more of it at dawn is that high caffeine intake would destroy the surefootedness that their daily tasks require

Which one of the following, if true, most seriously calls the anthropologists’ explanation into question?
(A) The drink is full of nutrients otherwise absent from the Akabe diet.
(B) The Akabe also drink the tea in the evening, after their day’s work is done.
(C) The leaves used for the tea contain a soluble narcotic.
(D) Akabe children are introduced to the tea in only a very weak form.
(E) When celebrating, the Akabe drink the tea in large quantities.

For me, the answer is D as it negates the reasoning that the Tea causes surefootedness but the answer seems to be is C and I don't know why!
gmat-admin's picture

Hi Jalaj,

I don't think you've accurately summarized the conclusion here.

The premise is that, although the Akabe people enjoy the tea, they avoid drinking too much.

The conclusion is that they avoid drinking too much tea BECAUSE high caffeine intake would destroy the surefootedness that their daily tasks require.

So, the conclusion is concerned with WHY the Akabe people don't drink more tea in the morning (especially since it tastes good).

Answer choice C suggests there's a different REASON for not drinking a lot of tea: the tea contains a narcotic
This certainly weakens the conclusion, because it provides an ALTERNATE REASON for not drinking a lot of tea.

Answer choice D does not provide an ALTERNATE reason for not drinking a lot of tea. It just tells us that children are given a weak form of the tea.

Does that help?

Cheers,
Brent

https://gmatclub.com/forum/images-from-ground-based-telescopes-are-invariably-distorted-by-the-208210.html
sir i cant figure out why d is wrong and e is correct
gmat-admin's picture

Citizens of Parktown are worried by the increased frequency of serious crimes committed by local teenagers. In response, the city government has instituted a series of measures designed to keep teenagers at home in the late evening. Even if the measures succeed in keeping teenagers at home, however, they are unlikely to affect the problem that concerns citizens, since most crimes committed by local teenagers take place between 3 pm and 6 pm.

Which of the following, if true, most substantially weakens the argument?

a) Similar measures adopted in other places have failed to reduce the number of teenagers on the streets in the late evening.

b) The crimes committed by teenagers in the afternoon are mostly small thefts and inconsequential vandalism.

c) Teenagers are much less likely to commit serious crimes when they are at home than when they are not at home.

d) Any decrease in the need for police patrols in the late evening would not mean that there could be more intensive patrolling in the afternoon.

e) The schools in Parktwon have introduced a number of after-school programs that will be available to teenagers until 6 pm on weekday afternoons.
gmat-admin's picture

Hi Brent,

Can you please help me with this CR

In order to gauge the productivity of employees, a particular call center uses the number of calls handled per hour as a measure. However, the number of calls handled per hour is not a correct measure to measure productivity since the number of queries effectively resolved is not being taken into consideration. Therefore, calls handled per hour is not the correct way to measure productivity of workers. Which of the following statements, if true, most weakens the above argument.

A) Call Center employees are representative of workers in general.
B) Call Center employees only redirect calls to the relevant department and do not answer queries.
C) Answering calls is the main activity of Call Center Employees.
D) All of the Above
E) None of the above
gmat-admin's picture

This question isn't very GMAT-like. What's the source?
An official CR question would never ask "Which of the following statements, if true, MOST weakens the above argument" and then have an answer choice that says "All of the above" (All of the answer choices MOST weaken the argument? :-)

For this reason, it's best to stick with Official GMAT practice CR questions, since they best represent what you'll find on test day.

You'll find 1000's of Official CR questions here: https://gmatclub.com/forum/gmac-official-guides-the-master-directory-lin...

Just click on an Official Guide and start answering CR questions.

That said, let's try to answer the question.

PREMISE: To measure productivity, center uses the number of calls handled per hour
PREMISE: The number of queries effectively resolved is not taken into consideration.
CONCLUSION: Calls handled per hour is not the correct way to measure productivity of workers.

As we check the answer choices, we must remind ourselves of the conclusion.

A) Call Center employees are representative of workers in general.
Does this weaken the conclusion that calls handled per hour is not the correct way to measure productivity?
NO. Eliminate A.

B) Call Center employees only redirect calls to the relevant department and do not answer queries.
Does this weaken the conclusion that calls handled per hour is not the correct way to measure productivity?
YES!
This means that the main premise (The number of queries effectively resolved is not taken into consideration) is irrelevant, since the Call Center Employees have nothing to do with whether the issue is resolved.
KEEP B

C) Answering calls is the main activity of Call Center Employees.
Does this weaken the conclusion that calls handled per hour is not the correct way to measure productivity?
NO. Eliminate C.

Answer: C

Cheers,
Brent

Hi Brent,

The below question is from GMAT prep test 4. Don't why the answer is C and not D.

Editorial: In Ledland unemployed adults receive government assistance. To reduce unemployment, the government proposes to supplement the income of those who accept jobs that pay less than government assistance, thus enabling employers to hire workers cheaply. However, the supplement will not raise any worker’s income above what government assistance would provide if he or she were not gainfully employed. Therefore, unemployed people will have no financial incentive to accept jobs that would entitle them to the supplement.

Which of the following, if true about Ledland, most seriously weakens the argument of the editorial?

A) The government collects no taxes on assistance it provides to unemployed individuals and their families.

B) Neighboring countries with laws that mandate the minimum wage an employer must pay an employee have higher unemployment rates than Ledland currently has.

C) At any given time, people who are currently employed have the best chance of being offered a job that will give them an income significantly greater than government assistance would give them.

D) The financial assistance that the government provides to people who have no other income is less than the average starting wage.

E) People sometimes choose a job for reasons that have nothing to do with the financial benefits it offers.
gmat-admin's picture

Question link: https://gmatclub.com/forum/editorial-in-ledland-unemployed-adults-receiv...

Here's an example of what the proposed supplement offers:
If government assistance is $500/month, and you accept a job that pays $300/month, then you'll receive a $200/month supplement to get you to the government assistance level of $500/month

CONCLUSION: Unemployed people will have no financial incentive to accept jobs that would ENTITLE THEM TO THE SUPPLEMENT.
NOTE: If you have a job AND are still entitled to the supplement, then that means your job must pay less than government assistance.

The problem with answer choice D is that it does not address the people specified in the conclusion. The conclusion concerns ONLY those people who are WORKING, but their jobs pay less than government assistance.
Answer choice D is about people who are NOT working at all.

On the other hand, answer choice C says that those working at a job (that pays less than government assistance) have a good chance of getting a job that pays MORE than government assistance. So, there IS a financial incentive to accept a low-paying job. Doing so increases your chances of getting a higher-paying job.

Does that help?

Cheers,
Brent

https://gmatclub.com/forum/although-fullerenes-spherical-molecules-made-entirely-of-78298.html
sir if some fullerenes is found on material other than earth doesnt it also weakens the question by opening up the possibility that structure won't match?
im confused b/w b and d
gmat-admin's picture

https://gmatclub.com/forum/archaeologists-seeking-the-location-of-a-legendary-siege-and-destructi-38845.html
sir, what is the conclusion here?
gmat-admin's picture

Question link: https://gmatclub.com/forum/archaeologists-seeking-the-location-of-a-lege...

There is no conclusion here. Instead we are asked "The force of the evidence cited above is most seriously weakened if which of the following is true?"

The main piece of EVIDENCE is that location of type 3 pottery found at the bottom of the middle layer.
The idea is that different layers will hold artifacts from different time periods.

Answer choice A hurts this notion, because gerbils can cause artifacts to "move" to lower layers.
This weakens the relationship between layers and time periods.

Does that help?

Cheers,
Brent

https://gmatclub.com/forum/in-january-there-was-a-large-drop-in-the-number-of-new-89404.html
please explain.

Hi Brent - can you provide your thoughts on the MoneyLand casino question? I understand why it is the answer after reading the other solutions, however, when reviewing the question myself, I X'ed that one quickly thinking it is Out-of-scope.

https://gmatclub.com/forum/with-seventeen-casinos-moneyland-operates-the-most-casinos-222382.html
gmat-admin's picture

Question link: https://gmatclub.com/forum/with-seventeen-casinos-moneyland-operates-the...

The question boils down to this sentence "To complete its acquisition of Eldorado, Apex must sell five casinos to comply..."

So, who will end up buying those 5 casinos?

Answer choice A tells us that Apex, Eldorado, and Moneyland are the ONLY organizations licensed to operate casinos in the state. So, there are only 3 organizations that can buy the 5 casinos.

Apex can't buy the 5 casinos, because they were told to sell them.
Eldorado can't buy the 5 casinos, because Eldorado is being acquired by Apex.
So, the only organization that can buy the 5 casinos is Moneyland.
If Moneyland buys the 5 casinos, it will then own 22 casinos, which will be more that the number of casinos that Apex will have. This contradicts the conclusion that Apex will have the most casinos in the state.

Does that help?

Cheers,
Brent

hi Brent need help to decode this one

The difference in average annual income in favor of employees who have college degrees, compared with those who do not have such degrees, doubled between 1980 and 1990. Some analysts have hypothesized that increased competition between employers for employees with college degrees drove up income for such employees.

Which of the following, if true, most seriously undermines the explanation described above?


(A) During the 1980s a growing percentage of college graduates, unable to find jobs requiring a college degree, took unskilled jobs.
(B) The average age of all employees increased slightly during the 1980s.
(C) The unemployment rate changed very little throughout the 1980s.
(D) From 1980 to 1990 the difference in average income between employees with advanced degrees and those with bachelor’s degrees also increased.
(E) During the 1980s there were some employees with no college degree who earned incomes comparable to the top incomes earned by employees with a college degree.
gmat-admin's picture

Question link: https://gmatclub.com/forum/the-difference-in-average-annual-income-in-fa...

Tough question! Here's my take:

CONCLUSION: The income difference doubled BECAUSE the demand for employees with college degrees increased.
In other words, between 1980 and 1990, employers were competing to hire employees with college degrees.

(A) During the 1980's a growing percentage of college graduates, unable to find jobs requiring a college degree, took unskilled jobs.

Wait! Unable to find jobs?

But I thought employers were competing to hire college graduates. If so, why are these college graduates unable to find jobs?
This certainly doesn't line up with the analysts' conclusion that income difference doubled BECAUSE the demand for employees with college degrees increased.

Does answer choice A explain WHY income difference doubled?
No, but answer choice A does weaken the conclusion regarding what actually caused the increase in income difference.

Does that help?

Cheers,
Brent

HI Brent. How many OG's do you suggest one should solve before Test day ?
gmat-admin's picture

My quick answer is "as many as it takes." :-)

The number of practice questions required depends on your present skills AND your target score.

For example, let's say someone needs a 550 score to get into a local MBA program. If that person finishes the CR module and can easily answer twenty 650-800 level CR questions in a row, then that person probably doesn't need to practice CR any more.

Conversely, let's say someone needs a 700+ score to get into top 10 MBA program. If that person finishes the CR module and can answer only 50% of the 350-500 level CR questions, then that person probably needs to practice CR a lot more.

So, in general, keep answering CR questions until you can answer questions in the difficulty range you are trying to achieve.

Does that help?

Cheers,
Brent

I just finished all the CR Videos upto video 6. Should I also do all the Weaken CR questions from 2019 OG and all others in one go?

OR

Should I do all the above questions and once done with all CR Videos then I shall Start from OG 2015, then Pick OG 2016, ...till OG 2019 in one go attempting all CR type questions regular touch?
gmat-admin's picture

At this point, I suggest you answer most (if not all) of the linked questions in the Reinforcement Activities box. Doing so will help you see all of the intricacies of this question type.

Later, if you find that you need to work on this question type, you'll find 100's of additional questions to practice with.

Cheers,
Brent

Politician: Hybrid cars use significantly less fuel per kilometer than nonhybrids. And fuel produces air pollution, which contributes to a number of environmental problems. Motorists can save money by driving cars that are more fuel efficient, and they will be encouraged to drive hybrid cars if we make them aware of that fact. Therefore, we can help reduce the total amount of pollution emitted by cars in this country by highlighting this advantage of hybrid cars.

Which of the following, if true, would most indicate a vulnerability of the politician's argument?

(A) People with more fuel-efficient cars typically drive more than do those with less fuel-efficient cars.
(B) Not all air pollution originates from automobiles.
(C) Hybrid cars have already begun to gain popularity.
(D) Fuel-efficient alternatives to hybrid cars will likely become available in the future.
(E) The future cost of gasoline and other fuel cannot be predicted with absolute precision or certainty.
-----------------------------------------------
OptE: WRONG! & Trap! We have to attack Conclusion and this attacks a premise instead.

Was this actually the reason Answer wasn't E but A because A counters the Conclusion ie Reduction of Pollution from Cars.

Question Link:
https://gmatclub.com/forum/politician-hybrid-cars-use-significantly-less-fuel-per-kilometer-202763.html
gmat-admin's picture

Question link: https://gmatclub.com/forum/politician-hybrid-cars-use-significantly-less...

We must remember the conclusion: We can help reduce car pollution by highlighting this advantage of hybrid cars.

A) If this premise is true, then the amount of pollution emitted might not change at all.

E) I guess E kind of attacks the premise that "motorists can save money by driving cars that are more fuel efficient" but regardless of the price of gas, hybrid vehicles will still save money. In fact, if the price goes up, motorists will save even more money.

Does that help?

Cheers,
Brent

The difficulty with the proposed high-speed train line is that a used plane can be bought for one-third the price of the train line, and the plane, which is just as fast, can fly anywhere. The train would be a fixed linear system, and we live in a world that is spreading out in all directions and in which consumers choose the free-wheel systems (cars, buses, aircraft), which do not have fixed routes. Thus a sufficient market for the train will not exist.

Which of the following, if true, most severely weakens the argument presented above?


(A) Cars, buses, and planes require the efforts of drivers and pilots to guide them, whereas the train will be guided mechanically.

(B) Cars and buses are not nearly as fast as the high-speed train will be.

(C) Planes are not a free-wheel system because they can fly only between airports, which are less convenient for consumers than the high-speed train's stations would be.

(D) The high-speed train line cannot use currently underutilized train stations in large cities.

(E) For long trips, most people prefer to fly rather than to take ground-level transportation.


I kept B and C
Where I noticed that C goes *against the premise*(as taught). The premise states: "consumers choose the free-wheel systems (cars, buses, aircraft), which do not have fixed routes."

And since C went against Premise by disapproving one means of transport. Therefore, I chose B as it brings in a new info/premise to the arguement to weaken the conclusion, and rest were clearly bad options or were strengthening.
gmat-admin's picture

Nice question.

The passage tells us that a sufficient market for the train will not exist.

The author supports this conclusion by telling us that plane are better than high-speed trains because they're cheaper, and because they aren't restricted to fixed routes. The passage also mentions that cars, buses and planes are free-wheel systems, because they can fly anywhere, and that this is better than the fixed routes of the train.

Answer choice B focuses its attention on cars and buses, while the original argument focuses mainly on planes versus high-speed trains. Also, answer choice B doesn't address the main aspect of the argument, which is that free-wheel is better than fixed route. Keep in mind that, even though cars and buses are slower, people might still prefer those modes of transportation, since they allow people to travel anywhere and not just along fixed routes.

Answer choice C, addresses the planes vs high-speed trains aspect of the argument AND it addresses the fixed route aspect. It says that, while planes can fly in any direction (which makes it FREE-WHEEL-ISH), they are still very limited in the fact that they can fly ONLY to airports. While this doesn't necessarily destroy the argument, it still weakens it.

Does that help?

Cheers,
Brent

Hi Brent,

I found your CR strategy is very helpful. I can pick the correct answers rather than before. However, assumptions take me a lot of time. Can I just think only one assumption or just skip it?

Pages

Office Hours

On December 20, 2023, Brent will stop offering office hours. 

Change Playback Speed

You have the option of watching the videos at various speeds (25% faster, 50% faster, etc). To change the playback speed, click the settings icon on the right side of the video status bar.

Have a question about this video?

Post your question in the Comment section below, and a GMAT expert will answer it as fast as humanly possible.

Free “Question of the Day” emails!